Math, asked by samadabdul16296, 5 days ago

Express in the form of p/q
12.53bar +0.23bar

Answers

Answered by hukam0685
24

Step-by-step explanation:

Given:

12. \overline{53} + 0.\overline{23} \\

To find: Express in the form of p/q

Solution:

Let

x = 12. \overline{53} + 0.\overline{23} \\ \\

Step 1: Remove bar

x = 12.535353... + 0.232323... \\

Step 2: Express x in terms of two constant

x = x_1 + x_2 \\  \\ x_1 = 12.535353... \\  \\ x_2 = 0.232323... \\

Step 3: Write x1 in terms of p/q

x_1 = 12.535353... \\  \\ 100x_1 = 1253.535353... \\  \\ subtract \: both \\  \\ 99x_1 = 1241 \\  \\ x_1 =  \frac{1241}{99}  \\  \\

Step 4: Write x2 in terms of p/q

x_2 = 0.232323... \\  \\ 100x_2 = 23.232323... \\  \\ subtract \\  \\ 99x_2 = 23 \\  \\ x_2 =  \frac{23}{99}  \\  \\

Step 4: Find x by adding both x1 and x2

x =  \frac{1241}{99}  +  \frac{23}{99}  \\  \\ x =  \frac{1241 + 23}{99}  \\  \\ x =  \frac{1264}{99}  \\  \\

Final Answer:

\bold{12. \overline{53} + 0.\overline{23} =  \frac{1264}{99}}  \\  \\

This way the given expression can be written in terms of p/q form.

Hope it helps you.

To learn more on brainly:

write 357.417417... in p/q form

https://brainly.in/question/9109261

Answered by MaheswariS
2

\underline{\textbf{Given:}}

\mathsf{12.\overline{53}+0.\overline{23}}

\underline{\textbf{To find:}}

\mathsf{\dfrac{p}{q}\;form\;of\;12.\overline{53}+0.\overline{23}}

\underline{\textbf{Solution:}}

\mathsf{Let\;x=12.\overline{53}+0.\overline{23}}

\mathsf{x=12.535353\;.\;.\;.\;.+0.232323\;.\;.\;.\;.}

\implies\mathsf{x=12.767676\;.\;.\;.\;.}----------(1)

\mathsf{100\,x=1276.767676\;.\;.\;.\;.}----------(2)

\mathsf{(2)-(1)\;\implies}

\mathsf{100x-x=1276-12}

\mathsf{99\,x=1264}

\implies\mathsf{x=\dfrac{1264}{99}}

\therefore\boxed{\mathsf{12.\overline{53}+0.\overline{23}=\dfrac{1264}{99}}}

\underline{\textbf{Find more:}}

Express 0.432323 in the form p/q, where p and q are integers and q not equal to 0

​https://brainly.in/question/38619483

Express the following in p/q form 0.19 bar​

https://brainly.in/question/38397623

Similar questions